Which of the following is an example of a positive economic statement? Question 19 options: a) Corrupt politicians ought to be voted out of office. b) An increase in the price of gasoline will cause a reduction in the amount of gasoline purchased. c) Marginal tax rates should be reduced for individuals in the highest tax bracket. d) Workers with families should be paid at least the minimum wage. e) If crime rates were reduced, the world would be a better place to live.

Answers

Answer 1
Its B

It has a cause and effect

Related Questions

Selected sales and operating data for three divisions of different structural engineering firms are given as follows: Division A Division B Division C Sales $ 12,000,000 $ 14,000,000 $ 25,000,000 Average operating assets $ 3,000,000 $ 7,000,000 $ 5,000,000 Net operating income $ 600,000 $ 560,000 $ 800,000 Minimum required rate of return 14% 10% 16% Required: 1. Compute the return on investment (ROI) for each division using the formula stated in terms of margin and turnover. 2. Compute the residual income (loss) for each division. 3. Assume that each division is presented with an investment opportunity that would yield a 15% rate of return. a. If performance is being measured by ROI, which division or divisions will probably accept or reject the opportunity

Answers

Answer:

1. See part 1 below for the calculations.

2. We have:

Division A's Residual Income (loss) = $180,000

Division B's Residual Income (loss) = ($140,000)

Division C's Residual Income (loss) = $0

3.a. Only Division B will probably accept the investment opportunity.

3.b. Divisions A and B will probably accept the investment opportunity.

Explanation:

Note: This question is not complete as the part 3-b of the requirement is omitted. The question is therefore completed before answering the question by providing the part 3-b as follows:

b. If performance is being measured by residual income, which division or divisions will probably accept the opportunity?

The explanation of the answer is now provided as follows:

The following are given:

                                              Division A        Division B           Division C

Sales                                     $12,000,000     $14,000,000     $25,000,000

Average operating assets    $3,000,000      $7,000,000        $5,000,000

Net operating income             $600,000          $560,000            $800,000

Min. req'd rate of return               14%                      10%                      16%

1. Compute the return on investment (ROI) for each division using the formula stated in terms of margin and turnover.

The relevant formulae are as follows:

Margin = Net Operating Income / Sales

Turnover = Sales / Average Operating Assets

Return on Investment = Margin * Turnover

Therefore, we have:

Division A:

Margin = $600,000 / $12,000,000 = 0.05, or 5%

Turnover = $12,000,000 / $3,000,000 = 4 times

Return on Investment = 5% * 4 = 0.20, or 20%

Division B:

Margin = $560,000 / $14,000,000 = 0.04, or 4%

Turnover = $14,000,000 / $7,000,000 = 2 times

Return on Investment = 4% * 2 = 0.08, or 8%

Division C:

Margin = $800,000 / $25,000,000 = 0.032, or 3.20%

Turnover = $25,000,000 / $ 5,000,000 = 5 times

Return on Investment = 3.2% * 5 = 0.16, or 16%

2. Compute the residual income (loss) for each division.

The following is the formula to use:

Residual Income (loss) = Net Operating Income - (Minimum Required Return * Average Operating Assets)

Therefore, we have:

Division A's Residual Income (loss) = $600,000 - (14% * $3,000,000) = $180,000

Division B's Residual Income (loss) = $560,000 - (10% * $7,000,000) = ($140,000)

Division C's Residual Income (loss) = $800,000 - (16% * $5,000,000) = $0

3-a. Assume that each division is presented with an investment opportunity that would yield a 15% rate of return. If performance is being measured by ROI, which division or divisions will probably accept the opportunity?

If a division's Return on Investment (ROI) is less than 15%, the decision criterion is to accept the investment opportunity. Otherwise, it will be rejected. Therefore, only Division B is will probably accept the investment opportunity, based on the results of Part 1 above. Division A and C will reject it.

3-b. Assume that each division is presented with an investment opportunity that would yield a 15% rate of return. If performance is being measured by residual income, which division or divisions will probably accept the opportunity?

The decision criterion is for a division to accept the investment opportunity if its minimum required rate of return is lower than 15%. Otherwise, it will be rejected.

Based on the information in the question, Divisions A and B will probably accept the investment opportunity. Division C will reject it.

A DuPont analysis is conducted using the DuPont equation, which helps to identify and analyze three important factors that drive a company’s ROE. According to the equation, which of the following factors directly affect a company’s ROE? Check all that apply. Total Assets / Total Common Equity Net Income / Sales Price per Share / Earnings per Share

Answers

Answer:

Total Assets / Total Common Equity

Explanation:

Depend upon theDu Pont Equation,

The following formula should be used  

ROE = Net profit margin × Total asset turnover × Equity multiplier

And,

ROE = (Net profit ÷ Sales) × (Sales ÷ Total Assets) × (Assets ÷ Equity)

So as per the above formula, the above answer should be considered  

An Uber driver faces costs for driving that include sunk costs like insurance that contribute $.50 to the average cost per mile. Yet when a rider offers to pay less than $0.50 per mile for a ride, the driver agrees because

Answers

Answer:

sunk costs like auto insurance (in this case) do not increase as driving increases

Explanation:

In the case when the uber driver faces cost for driving so the sunk cost such as insurance that contribute $0.50 but the other rider pay lower than $0.50 per mile so here the driver agrees as the sunk cost would not increased in the same way like driving rises.

Therefore the above represent the answer

The master budget includes individual budgets for sales, production or merchandise purchases, various expenses, capital expenditures, and cash.
A. True
B. False

Answers

Answer: True

Explanation:

The statement that the master budget includes the individual budgets for the sales, production or merchandise purchases, various expenses, capital expenditures, and cash is true.

It should be noted that the master budget consists of the budgets of the lower-level that can be found in an organization, the cash flow forecasts, and a financial plan.

The master budget gives firm a wider view of its finances and is used in making organizational decisions.

n economy has three industries, farming, building, and clothing. For every dollar of food produced, the farmers use $0.1, the builders use $0.05, and the tailors use $0.05. For every dollar of building, the builders use $0.05, the farmers use $0.11, and the tailors use $0.13. For every dollar of clothing produced, the tailors use $0.06, the builders use $0.15, and the farmers use $0.1. If the external demand for food is $260 million, for building is $200 million, and for clothing is $120 million, what should be the total production for each industry

Answers

Answer:

Please find the complete question for the solution:

Explanation:

Using formula:

[tex]\text{Production for each industry = External demand} \div \%\ \text{to meet external demand}[/tex]

[tex]- \ \ \ \ \ \ \ \ \ \ Farming \ \ \ \ \ \ \ \ \ \ Building \ \ \ \ \ \ \ \ \ \ Clothing \ \ \ \ \ \ \ \ \ \ \text{Remaining \% to meet external demand}\\\\\text{For every \$ of:}\\\\\\ Food \ produced \ \ \ \ \ \ 7\% [(\frac{\$0.07}{\$1})\times 100] \ \ \ \ \ \ 3\% [(\frac{\$0.03}{\$1})\times 100] \ \ \ \ \ \ \ 6\% [(\frac{\$0.06}{\$1})\times 100 \ \ \ \ \ 84\%\\\\[/tex]

[tex]Building\ \ \ \ \ \ \ \ 5\% [(\frac{\$0.05}{\$1})\times 100] \ \ \ \ \ \ \ \ 5\ [\frac{\$0.05}{\$1})\times 100] \ \ \ \ \ \ \ \ \ \ 13\% [(\frac{\$0.13}{\$1})\times 100]\ \ \ \ \ \ \ \ \ \ 77\%\\\\Clothing\ produced \ \ \ 18\% [(\frac{\$0.18}{\$1})\times 100]\ \ \ 4\% [(\frac{\$0.04}{\$1})\times 100]\ \ 13\% [(\frac{\$0.13}{\$1})\times 100] \ \ 65\%\\\\[/tex]

So, the answer is:

[tex]\text{Farming production} (\$170\ mil \div 84\%)\ \ \ \ \ \ \ \ \ \ \ \ \ \ \ \ \ \ \$202.38\ million\\\\\text{Building production} (\$140\ mil \div 77\%)\ \ \ \ \ \ \ \ \ \ \ \ \ \ \ \ \ \ \$181.82 \ million\\\\\text{Clothing production} (\$240\ mil \div 65\%)\ \ \ \ \ \ \ \ \ \ \ \ \ \ \ \ \ \ \$369.23\ million\\\\[/tex]

The total production for each industry is as follows:

                               Food       Building      Clothing

                             (Million)     (Million)       (Million)

Total production   $325      $281.69       $173.91

How is total production determined?

The total production for each industry is determined by dividing the external demand by external usage per dollar.

The external usages per dollar are the differences between what the primary producers consumed or used and the total production per dollar.

Data and Calculations:

                                Food        Building        Clothing

Farmers' usage        $0.1         $0.11                $0.10

Builders' usage     $0.05         $0.05             $0.15

Tailors' usage       $0.05          $0.13              $0.06

External usage     $0.80         $0.71               $0.69

External demand  $260         $200              $120

Total production $325          $281.69        $173.91 ($120/$0.69)

Thus, the total production for each industry is Food, $325 million, Building, $281.69 million, and Clothing, $173.91 million.

Learn more about production and consumption https://brainly.com/question/4978509

Find the present value of $19,000 in 11 months at 5.1% interest

Answers

Answer:

$19,886.396

Explanation:

Given :

Interest rate = 5.1% = 5.1

Principal = $19000

Period = 11 months = (11/12)year

The present value of 19000 in 11 months at 5.1% interest Can be obtained using the relation:

PV = P(1 + r)^n

PV = 19000(1 + 0.051)^(11/12)

PV = 19000(1.051)^(11/12)

PV = 19000 * 1.0466524

PV = 19886.396

Hence, the present value is $19,886.396

Mass customization is ______. Group of answer choices the redesign of components found to be faulty in earlier versions of a product the mass production of individually customized products the reuse of components discarded from scrapped products the production of each product by hand for specific custom

Answers

Answer:

production of individually customized products

Explanation:

Mass customization can be described as when a company produces and delivers market goods and services that are suited to meet the needs of individual customers

It combines the benefit of low cost associated with mass production with the customization of goods.

An example of a product that is mass customized is the mobile phone. A mobile phone is mass produced but due to its software, users are able to modify or customize their phone to suit their needs

Types of Mass customization

Collaborative customization:  In this type of Mass customization, customers and the company work together to create a good or service that meets the unique needs of the consumer.  Adaptive customization: the good or service created can be further customized by the consumer to suit their needs.  Transparent customization: unique products are made for each consumer .  Cosmetic customization: different types of standardized products are made for various groups of customers.

Advantages of Mass customization

Customer satisfaction increasesGoods are produced at lower costs

Disadvantages of Mass customization

It would be difficult for the company to build up stock ahead of time due to the unique needs of the customersthere would be an increased wait time from the time the order is made till when it is delivered

Hot Shot Delivery Inc. provides the following year end data:
2020 2019
Cash $65,000 $38,000
Accounts Receivable 60,000 39,000
Merchandise Inventory 66,000 26,000
Property, Plant & Equip 219,000 154,000
Total Assets 410,000 257,000
Sales Revenue $530,000
Cost of Goods Sold 180,000
Interest Expense 30,000
Net Income 112,000
Calculate the rate of return on total assets for 2018.
a. 55.3%.
b. 52.5%.
c. 42.6%.
d. 27.3%.

Answers

Answer:

c. 42.6%

Explanation:

Average total assets = $410,000+$257,000/2

Average total assets = $667,000

Average total assets = $333,500

Net income = $112,000

Interest expenses = $30,000

Return on total assets = Net income + Interest expenses / Average total assets

Return on total assets = $112,000 + $30,000 / $333,500

Return on total assets = 0.42388060

Return on total assets = 42.39%

Assuming no interactions, the main effects analysis of a one-half fractional factorial experiment compared to a comparable full factorial experiment yields which of the following outcomes?

a. Only the full factorial is accurate
b. The one-half factorial gives one-half of the final outcome
c. Both can be calculated but computer software is required
d. Both results are approximately the same

Answers

Answer:

d. Both results are approximately the same

Explanation:

As there is no interaction

One-half factorial will give half of the result of the experiment.

When there is a lack of resources, we also do the one-half factorial to complete the full factorial experiment.

Hence, both provide the approximately same outcomes of the experiment.

What is the initial selling price needed to obtain a target profit of $50,000 using the manufacturing cost markup method

Answers

Answer:

$15 per unit

Explanation:

Note "The full question is attached as picture below"

Markup = (Fixed Costs + Desired Profit) / Units produced

Markup = ($300,000 + $150,000 + $50,000) / 50,000

Markup = $500,000 / 50,000 units

Markup = $10 per unit

Variable cost per unit = Variable cost / Quantity

Variable cost per unit = ($200,000 + $50,000) / 50,000

Variable cost per unit = $5 per unit

Sales price = Variable cost + Markup

Sales price = $5 per unit + $10 per unit

Sales price = $15 per unit

So, the initial selling price needed to obtain a target profit of $50,000 using the manufacturing cost markup method is $15 per unit.

Because this market is a monopolistically competitive market, you can tell that it is in long-run equilibrium by the fact that at the optimal quantity for each firm. Furthermore, the quantity the firm produces in long-run equilibrium is the efficient scale. True or False: This indicates that there is a markup on marginal cost in the market for shirts. True

Answers

Answer:

Because this market is a monopolistically competitive market, you can tell that it is in long-run equilibrium by the fact that P = ATC, P>ATC, MR =MC, or MR>MC at the optimal quantity.  Furthermore, the quantity the firm produces in long-run equilibrium is the efficient scale. True False

This indicates that there is a markup on marginal cost in the market for shirts. True False

Explanation:

In the long run, monopolistically-competitive entities produce at a level where marginal cost and marginal revenue are equal. This makes it impossible for individual companies to sell their products at prices above the average cost. This situation means that monopolistically-competitive companies will always earn zero economic profit in the long run.

The world's average (mean) GDP per capita is $10,515. There are roughly 7 billion people in the world. About 20% of the world's population produces 50% of the world's total GDP. How much GDP does the top 20% produce

Answers

Answer:

$36.802.50 billion

Explanation:

GDP per capita measures the standard of living of the people in a country. The higher the GDP per capita, the higher the standard of living

GDP per capita = GDP / population  

Gross domestic product is the total sum of final goods and services produced in an economy within a given period which is usually a year

GDP calculated using the expenditure approach = Consumption spending by households + Investment spending by businesses + Government spending + Net export

$10,515 = GDP / 7 billion

GDP = $10,515 x 7 billion

GDP = 73,605 billion

GDP of the top 20% = 50% x 73,605 billion = $36.802.50 billion

Suppose you have $100 of endowment, and you are offered a chance to buy a lottery which costs $36. The lottery has 43% of chance to win a prize of $G, or you just lose and get nothing. Suppose your utility function on wealth is u(w)=w^1/2. What is the least prize size G that you will be willing to buy the lottery? (Round to the second digit after decimal point.)

Answers

Answer:

96.02

Explanation:

Lottery's Expected utility = [tex]\sqrt{100}[/tex] = 10

Income in good state = 100 - 36 + G = 64 + G

Income in bad state = 100 - 36 = 64

Probability in good state = 43%

Probability in bad state = 100% - 43% = 57%

Expected utility = Probability in good state x [tex]\sqrt{(64 + G )}[/tex] + Probability in bad state x [tex]\sqrt{64\\}[/tex]

10 = 43% x [tex]\sqrt{(64 + G )}[/tex] + 57% x 8

10 = 43% x [tex]\sqrt{(64 + G )}[/tex] + 4.56

10 - 4.56 = 43% x [tex]\sqrt{(64 + G )}[/tex]

5.44 = = 43% x [tex]\sqrt{(64 + G )}[/tex]

5.44 / 43% = [tex]\sqrt{(64 + G )}[/tex]

12.65 = [tex]\sqrt{(64 + G )}[/tex]

[tex]12.65^{2}[/tex] = [tex](\sqrt{(64 + G )})^{2}[/tex]

160.0225 = 64 + G

G = 160.0225 - 64

G = 96.0225

G = 96.02

On November 1, clients of Great Designs Company prepaid $4,250 for services to be provided in the future at a rate of $85 per hour. a. Journalize the receipt of cash. If an amount box does not require an entry, leave it blank. Nov. 1 fill in the blank 15e2fafaf020002_2 fill in the blank 15e2fafaf020002_3 fill in the blank 15e2fafaf020002_5 fill in the blank 15e2fafaf020002_6 b. As of November 30, Great Designs shows that 15 hours of services have been provided on this agreement. Journalize the necessary adjusting entry. If an amount box does not require an entry, leave it blank. Nov. 30 fill in the blank 55e33803103f004_2 fill in the blank 55e33803103f004_3 fill in the blank 55e33803103f004_5 fill in the blank 55e33803103f004_6 c. Determine the total unearned fees in hours and dollars at November 30. Unearned fees in dollars $fill in the blank b5fba80a1040fa8_1 Unearned fees in hours fill in the blank b5fba80a1040fa8_2 hours

Answers

Answer:

Total unearned fees in dollars at November 30 = $2975

Total unearned fees in hours at November 30 = 35 hours

Explanation:

The price elasticity of demand measures: Group of answer choices how responsive consumers are in the quantity they want when consumer incomes change how responsive producers are in the quantity they produce when the price changes how responsive consumers are in the quantity they want when the price changes how responsive producers are in the quantity they produce when consumer incomes change

Answers

Answer:

how responsive consumers are in the quantity they want when the price changes

Explanation:

The price elasticity of demand is

= Percentage change in quantity demanded ÷ percentage change in demand

So based on the above formula it shows that the consumers are responsive with regard to the quantity they need at the time when the price is changed

Therefore the above represent the answer

Answer:

Price

Inelastic

Elastic

Explanation:

got it right on edg

If Medicaid is expanded to cover a greater percentage of the population: the public debt will immediately increase. implicit liabilities will increase. implicit liabilities will decrease. implicit liabilities will be unaffected.

Answers

Answer: implicit liabilities will increase.

Explanation:

Implicit liabilities are incurred by government as a result of them having to take care of their citizens. Medicaid is one such liability.

If the government were to expand the percentage of people in the country that are to be covered by medical aid, this would mean that more Medicaid will be paid by the government which means that the implicit liabilities will increase.

Differentiate between a defined contribution pension plan and a defined benefit pension plan. Explain how the employer's obligation differs between the two types of plans.

Answers

Answer:

Differences Between a Defined Contribution Pension Plan and a Defined Benefit Pension Plan.

With a defined contribution pension plan, the benefit that will accrue to the employee is not known or defined ahead of her retirement.  But the contributions that will be made by the employer and the employee to fund the pension are clearly spelt out.

With a defined benefit pension plan, the benefit (i.e. the monthly payment to the retiree) is stated ahead of the pension time.  It is based on the employee's tenure and salary.  Employees do not contribute to the plan but are entitled to lifetime monthly payments.

Explanation:

The employer and each employee contribute some certain percentages to each worker's individual retirement account (IRA) under the defined contribution pension plan.  Under the defined benefit pension plan, the employer is solely responsible for funding the plan and the employee benefits via a monthly payment from the funding plan during retirement.

Financial statement data for the years 20Y5 and 20Y6 for Black Bull Inc. follow:
20Y5 20Y6
Net income $1,508,000 $2,676,000
Preferred dividends $60,000 $60,000
Average number of
common shares outstanding 80,000 shares 120,000 shares
a. Determine the earnings per share for 20Y5 and 20Y6.
b. Is the change in the earnings per share from 20Y5 to 20Y6 favorable or unfavorable?

Answers

Answer and Explanation:

a. The computation of the earning per share is given below:

As we know that

Earning per share = (Net income - preference dividend) ÷ (average no of common shares oustanding)

For 20Y5

= ($1,508,000 - $60,000)  ÷ 80,000 shares

= $18.1

For 20Y6

= ($2,676,000 - $60,000) ÷ 120,000 shares

= $21.8

b. Since the earning per share is increased from 20Y5 to 20Y6 so it is favorable

How jse reported the negative impact of the coronavirus on the economic conditions​

Answers

Answer:

C

Explanation:

sorry if im wrong tried my best

Your project is split into two teams across two different continents. They understand the work to be completed, as well as communication processes. But they still often argue about how the work should be accomplished and who should make decisions. Given what you know, what is the most likely cause for these issues?​

Answers

Answer:

Your project is split into two teams across two different continents. They understand the work to be completed, as well as communication processes. But they still often argue about how the work should be accomplished and who should make decisions. Given what you know, what is the most likely cause for these issues?

cultural differences

Explanation:

Future Value of Multiple Annuities Assume that you contribute $150 per month to a retirement plan for 20 years. Then you are able to increase the contribution to $350 per month for another 20 years. Given an 8 percent interest rate, what is the value of your retirement plan after 40 years

Answers

Answer:

$641,455.26

Explanation:

Calculation to determine the value of your retirement plan after 40 years

First step is to determine FV Using financial calculator

N = 40*12 = 480

I = 8%/12 = .6667

PV = 0,

PMT = $150

CPT FV =$523,651.17

N = 20*12 = 240

I = 8%/12 = .6667

PV = 0

PMT = $200 ($350 - $150)

CPT FV =$117,804.08

Now let determine the value of your retirement plan after 40 years

Sum of FV =$523,651.17+$117,804.08

Sum of FV =$641,455.26

Therefore the value of your retirement plan after 40 years will be $641,455.26

You notice that the price of Blu-ray players falls and the quantity of Blu-ray players sold increases. You suspect that _____ Blu-ray players shifts to the _____. demand for; left supply of; left demand for; right supply of; right

Answers

Answer:

supply of; right

Explanation:

When the supply curve shifts rightward, there would be a rightward shift of the supply curve. As a result of the rightward shift, supply would increase and the price falls.

When the price of a good falls, the quantity demanded increases. This is in line with the law of demand.

According to the law of demand, the higher the price, the lower the quantity demanded and the lower the price, the higher the quantity demanded.

Thus, when the price of blue ray players fall, there would be an increase in the quantity of demanded. there would a movement down along the demand curve.

Do we have to show repair expense of abnormal loss in consignment account.If yes,how ?​

Answers

Answer:

yes

Explanation:

to see the income and costs of the business

Bloom Company management predicts that it will incur fixed costs of $251,000 and earn pretax income of $365,100 in the next period. Its expected contribution margin ratio is 61%. Required: 1. Compute the amount of total dollar sales. 2. Compute the amount of total variable costs

Answers

Answer and Explanation:

The computation is shown below;

a. The amount of the total dollar sales is

Pretax income  = Sales value - Variable cost - Fixed cost

where,  

Sales value - variable cost = Contribution margin

$365,100  = Contribution margin - $251,000          

So,  

Contribution margin = $616,100

Now  

Contribution margin = Sales value × Contribution margin ratio

$616,100 = Sales value ×  61%

So,

Sales value = $1,010,000

b. The total variable cost is

= Sales - fixed cost - pre tax income

= $1,010,000 - $251,000 - $365,100

= $393,900

The journal entry to record the accrual of factory utilities is to: Multiple choice question. debit Factory Overhead and credit Utilities Payable debit Utilities Payable and credit Factory Overhead debit Factory Overhead and credit Cash debit Cash and credit Factory Overhead

Answers

Answer:

debit Factory Overhead and credit Utilities Payable

Explanation:

The journal entry to record the accrual of factory utilities is to: Debit Expense Account -  Factory Overhead and Credit Liability Account -Utilities Payable.

There is a bond that has a quoted price of 103.203 and a par value of $2,000. The coupon rate is 6.81 percent and the bond matures in 23 years. If the bond makes semiannual coupon payments, what is the effective annual interest rate

Answers

Answer: 6.53%

Explanation:

You can use an Excel worksheet to solve this.

The bond pays semi-annual coupons so you need to convert these measures to periodic measures:

Coupon = 6.81% * 2,000 * 0.5 = $68.10

Maturity term = 23 * 2 = 46 semi annual periods

Present value = 2,000 * 103.203 quoted price = $2,064.06

Use the Rate function as attached:

Periodic rate = 3.2694%

Annual rate = 3.2694 * 2

= 6.53%

considers the problem of building railway tracks under the assumption that pieces fit exactly with no slack. Now consider the real problem, in which pieces don’t fit exactly but allow for up to 10 degrees of rotation to either side of the "proper" alignment. Explain how to formulate the problem so it could be solved by simulated annealing

Answers

Answer:

By using simulated annealing we will sample the next state, evaluate and take the next state according to the probability e^Δv

Value function ( V ) = ( a * number of gaps ) + ( b * number of misconnected pieces ) + ( c * sum of sizes of gaps )

a,b,c = adjustable

Explanation:

In order to solve this problem by simulated annealing

First condition : assuming that pieces of the railways tracks fit exactly with no slack

Assume a state configuration of 32 pieces, use of discrete operations whose function is to remove pieces and reconnect it somewhere else without slack , we will also consider a continuous operations to help change angles to real values

Second condition : considering a real problem

This condition can be considered to be a closed loop because when one joint is moved all other joints are moved, here we will consider using a heuristic function

By using simulated annealing we will sample the next state, evaluate and take the next state according to the probability e^Δv

Value function ( V ) = ( a * number of gaps ) + ( b * number of misconnected pieces ) + ( c * sum of sizes of gaps )

a,b,c = adjustable

National Furniture Company has 25,000 shares of cumulative preferred 2% stock, $75 par and 200,000 shares of $10 par common stock. The following amounts were distributed as dividends: Year 1 $25,000 Year 2 88,000 Year 3 95,500 Determine the dividends per share for preferred and common stock for each year. If an answer is zero, enter '0'. Round all answers to two decimal places.

Answers

Answer:

Year 1

Preferred Dividend = $25,000

Common Stock Dividend  = $0

Year 2

Preferred Dividend = $37,500

Common Stock Dividend  = $50,500

Year 3

Preferred Dividend = $25,000

Common Stock Dividend  = $70,500

Explanation:

The dividends per share for preferred and common stock for each year.

Preferred Dividend

Is a fixed charge. When it is cumulative, all dividends in arrears are accumulated an paid in future when funds become sufficient before other dividends are paid.

Preferred Dividend = 25,000 x $75 x 2 % = $37,500

Common Stock Dividend

Holders of Common Stock receive their dividends after the Preferred Stock holders have received their dividends.

Calculations

Year 1

Preferred Dividend = $25,000 (owing $12,500)

Common Stock Dividend  = $0

Year 2

Preferred Dividend = $25,000 + $12,500 (owing ) = $37,500

Common Stock Dividend  = $88,000 - $37,500 = $50,500

Year 3

Preferred Dividend = $25,000

Common Stock Dividend  = $95,500 - $25,000 = $70,500

a T-bill money market fund that yields a sure rate of 4.6%. The probability distributions of the risky funds are: Expected Return Standard Deviation Stock fund (S)166%Bond fund (B)70%The correlation between the fund returns is 0.0800. What is the Sharpe ratio of the best feasible CAL

Answers

Answer:

The Sharpe ratio of the best feasible CAL is 0.3167.

Explanation:

Note: This question is not complete as some data are omitted. The complete qustion is therefore provided before answering the question. See attached pdf file for the complete question.

Risk-free rate = Sure rate of the T-bill money market fund = 4.6%, or 0.046

Sharpe ratio of a fund = (Expected return - Risk-free rate) / Standard Deviation …………. (1)

Using equation (1), we have:

Sharpe ratio of Stock fund = (16% - 4.6%) / 36% = 0.3167

Sharpe ratio of Bond fund = (7% - 4.6%) / 30% = 0.08

Since Sharpe Ratio of Stock Fund (S) is higher than Sharpe Ratio of Bond Fund (B), this implies that the best option is to invest in Stock Fund (S).

Therefore, the Sharpe ratio of the best feasible CAL is 0.3167.

Given the points (-4,8) and (6,-12)

Answers

Answer:

N/A

Explanation:

What is the question. There is nothing to answer.

where is the question please put the question so we can help
Other Questions
A Three-Point TurnChapter 1"You know, hardly anyone ever needs to do a three-point turn anymore," said Justin, trying to help Becky calm down."Oh, so it's not a useful skill AND I am probably going to fail the driving test because I can't do it anyway," Becky said, raising her voice for emphasis. "That should make me feel like a million bucks when I flunk."Justin was riding with Becky so she could take her driving test. He had volunteered for the job because he thought she would be less nervous with him than with their mom, but so far, he wasn't sure he was making any difference."Slow down, your turn is coming up here," he said, looking ahead."I know, I know," she replied, "I've been here before rememberthe last time I flunked."Justin was pretty sure if he had let her miss the turn, things would only have deteriorated further, but he wasn't sure he was fond of being the scapegoat for Becky's anxiety."Listen, you need to take a few deep breaths," he said, hoping he could help her at least relax a bit. "Being nervous won't help you with the three-point turn or anything else you have to do. Hey, did you just take that turn without your turn signal on?" This was going to be harder than he thought."Stop yelling at me," Becky replied, clearly frustrated, "I can't concentrate.""Look, you need to stop and get yourself together here," Justin started. "It is not just about passing the driving test. I don't want to get in an accident, so pull into that parking lot."Becky drove into the office building's parking lot where Justin was pointing. Justin knew they were less than a mile from the licensing office, and if she continued in this condition, he'd be having this same discussion three months from now when she tried the test again for the third time."You need to get a grip," he started after she put the car in park, "because you have studied and practiced driving all year. You know this stuff inside and out, backwards and forwards. What are you so nervous about?""I don't know, I don't know," Becky wailed, resting her head on the steering wheel. "I just get so tired of failing."Listening quietly as Becky sobbed, Justin realized this was about much more than a driving test. He also knew if he didn't find a way to help Becky things would just get worse.Chapter 2Justin took a deep breath and collected his thoughts. Becky was an unbelievably consistent straight-A student. It was Justin who got the bad grades in school, and Justin who had to repeat every math class he'd ever taken. It was Justin who wished he could get the grades Becky got. Some things came easier for Justin: He was athletic, handy with tools, and good at making the best of whatever life threw at him. Mom called him her "lemons into lemonade" kid. But for the most part, Becky succeeded easily, whereas Justin had to work and work to just get a passing grade.Rather than having Becky catalogue all the things she supposedly "failed" at, Justin decided to try an alternative approach, one that wouldn't remind him of all the ways he had failed."Okay, Becky, let's assume for a moment you fail this test again. What is the worst thing that could happen?" he asked."I would be the oldest kid at school without a license and be humiliated," she replied. Justin thought he heard a bit of panic in her voice but continued with his plan."Yes, but won't we still have to drive to school together for at least one more year anyway?" he asked."Yes, but..." she started."And who will know, if you don't tell anyone except your friends, that you don't have your license? You know Mom can't afford another car just for you, right?""Yes," she said quietly."So what difference does it make, really," he said. "Another three months to wait in the grand scheme of your life doesn't seem like all that long, right?""I suppose not," she said.Justin could tell she was breathing more slowly now. "Besides," he said, "I would miss all the practice driving with you," and for good measure he reached over and pinched her arm."Ow," she said, hitting back at him, "that hurt.""So let's go do this, okay?"Okay," she said. Becky cranked up the car, backed slowly out of the parking spot and drove up to the parking lot's exit. Justin noticed, as they waited for the traffic to clear, that she had remembered the turn signal.Read this line from Chapter 1:"Stop yelling at me," Becky replied, clearly frustrated, "I can't concentrate."Which of the following has a more negative connotation than frustrated? Concerned Distraught Puzzled Reassured Which of the following terms can be combined in this expression?13 minus x over 5 + 6.2 y + 20.5 x minus 3 over 8 yNegative 13 can be combined with 20.5.6.2 y can be combined with 20.5 x.6.2 y can be combined with Negative 3 over 8 y.Negative x over 5 can be combined with Negative 3 over 8 y. How many centimeters are there in 0.5 inches? help my homework pls. Use the grouping method to factor this polynomial.x^3 + 2x^2 + 12x +24A. (x^2+6)(x+2)B. (x^2 +2)(x+6)C. (x^2 +2)(x+12)D. (x^2 +12)(x+2) Quick Cleaners, Inc. (QCI), has been in business for several years. It specializes in cleaning houses but has some small business clients as well.a. Issued $21,000 of QCI stock for cash. b. Incurred $840 of utilities costs this month and will pay them next month. c. Paid wages for the current month, totaling $2,600. d. Performed cleaning services on account worth $3,800. e. Some of Quick Cleaners equipment was repaired at a total cost of $300. The company paid the full amount at the time the repair work was done. Required:Prepare journal entries for the above transactions, which occurred during a recent month. When species become challenging to classify, what is the most accurate technology for their identification? DNA sequencing DNA electrophoresis DNA amplification DNA barcoding DNA replication How does the following portion of Susan B. Anthony's speech support herargument?The preamble of the Federal Constitution says:We, the people of the United States, in order to form a moreperfect union, establish justice, insure domestic tranquility,provide for the common defense, promote the generalwelfare, and secure the blessings of liberty to ourselves andour posterity, do ordain and establish this Constitution forthe United States of America.It was we, the people; not we, the white male citizens; norwe, the male citizens; but we, the whole people, who formedthe Union. And we formed it, not to give the blessings ofliberty, but to secure them; not to the half of ourselves andthe half of our posterity, but to the whole people - womenas well as men. And it is a downright mockery to talk towomen of their enjoyment of the blessings of liberty whilethey are denied the use of the only means of securing themprovided by this democratic-republican government - theballot.O A. It proves that all citizens of the U.S. are given the same rights.OB. It shows Anthony should not be allowed to vote.O c. It shows only male citizens are addressed in the U.S.Constitution.D. It shows that most historical documents fail to addresswomen's rights. The sales price of a product is $100 per unit; the variable cost is $20 per unit; and fixed costs total $800. How many units must be sold to break even? Some service providers create (or at least predict) demand for certain outcomes, thus forming a specific target for their services. B It's important that the service provider do all that's necessary to fully understand the consumer's needs and expectations. C The levels of frankness and mutual understanding between the service provider and consumer go a long way in determining how well (or poorly) the service provider understands the consumer's desired outcomes. D An outcome is a tangible or intangible deliverable of an activity If XU = 3y-5 and RT = 32, find the value of y plzz help i only have like 5 minssssssWhat is the volume of this rectangular prism if an edge of each cube is 1 cm? 3,15,75,375 9th term HEY CAN SOMEONE HELP ME FAST I GIBE BRAINLIST Ilarawan ang katangian ni Kapitan Arnold at Kapitan Berting bilang mga pinuno ng barangay. The graph shows the function f(x) = |x h| + k. What is the value of h?h = 3.5h = 1.5h = 1.5h = 3.5 Find value a when the angle between vectors U=(2,a) and V = (-6;8) is pie/4 Biggest oil fields in the world? A negatively charged object is placed beside a negatively charged object What is the 100th term of the sequence with a1=222 and d=-5